What Do You Know About Hypertension Disease?

Approved & Edited by ProProfs Editorial Team
The editorial team at ProProfs Quizzes consists of a select group of subject experts, trivia writers, and quiz masters who have authored over 10,000 quizzes taken by more than 100 million users. This team includes our in-house seasoned quiz moderators and subject matter experts. Our editorial experts, spread across the world, are rigorously trained using our comprehensive guidelines to ensure that you receive the highest quality quizzes.
Learn about Our Editorial Process
| By Cardtrick
C
Cardtrick
Community Contributor
Quizzes Created: 9 | Total Attempts: 14,239
Questions: 30 | Attempts: 584

SettingsSettingsSettings
Hypertension Quizzes & Trivia

This quiz is an introduction to hypertension and some of the agents for treatment and control. As you will see, when you progress through disease states in cardiology, these agents are utilized repeatedly.


Questions and Answers
  • 1. 

    Which of the following is the scenario for most individuals with hypertension?

    • A.

      Reduced cardiac output

    • B.

      Elevated cardiac output

    • C.

      Reduced total peripheral resistance

    • D.

      Elevated total peripheral resistance

    Correct Answer
    D. Elevated total peripheral resistance
    Explanation
    For individuals with hypertension, the biggest issue is that the total peripheral resistance is elevated, not reduced. The resistance can be elevated due to a number of factors, but a common one is when arteries become occluded with plaque and cannot expand (atherosclerosis). Cardiac output rarely increases in a resting hypertensive individual, but it does happen in the short-term until metabolic autoregulation occurs.

    Rate this question:

  • 2. 

    Which of the following is a result of primary aldosteronism?

    • A.

      Hyperkalemia

    • B.

      Decreased sodium absorption

    • C.

      Increased water retention

    • D.

      Decreased extracellular fluid volume

    Correct Answer
    C. Increased water retention
    Explanation
    Primary aldosteronism is condition where there is an an elevation in the secretion of aldosterone by the adrenal cortex; there is too much aldosterone. In these situations, the effects of aldosterone will be more pronounced. So, when thinking of this question, it is important to know what the effects of aldosterone are.

    Aldosterone will work on the Na-K-ATPase pumps in the renal tubule. When the kidney senses too much sodium into the urine, it will do whatever it can to take it back into the body. Water will come with it, too. Unfortunately, potassium will be lost in exchange for the sodium picked up. Therefore, with too much aldosterone, there will be too much activity by the Na-K-ATPase pump, more sodium (and thus more water) will be reabsorbed, and more potassium will be excreted. This is how the kidney will try to keep sodium and water. Looking at the answer choices, we can rule out why some are wrong.

    (A) is incorrect. More potassium will be lost due to too much aldosterone and will result in HYPOkalemia.

    (B) is incorrect because more sodium will be reabsorbed. That is the point of aldosterone.

    (D) is incorrect because as more sodium is reabsorbed, more water will come with it and actually increase the extracellular fluid volume. Retaining water is important for the body, especially in dire situations.

    Therefore, (C) is correct. Water retention will have to increase because more sodium will be reabsorbed and thus more water will be reabsorbed.

    Rate this question:

  • 3. 

    What is the effect on serum potassium due to aldosterone?

    • A.

      Increase

    • B.

      Decrease

    • C.

      No effect

    Correct Answer
    B. Decrease
    Explanation
    Aldosterone will stimulate the Na-K-ATPase pump in the renal tubules. This is the mechanism to conserve sodium, which is highly important. Unfortunately, this also means that potassium has to be exchanged for this, which can result in HYPOkalemia. This is the result of when the renin-angiotensin system (RAS) is activated. Actually, some blood pressure medications that allow sodium to be excreted could result in the RAS being activated so that sodium will not be as much excreted. Again, this will force potassium out. This is why some individuals will take potassium supplements alongside certain blood pressure medications.

    Rate this question:

  • 4. 

    Which of the following is false regarding beta-blockers?

    • A.

      Heart rate decreases

    • B.

      Renin release decreases

    • C.

      At higher doses, they become more non-selective

    • D.

      They reduce exercise performance

    • E.

      All of the following are true

    Correct Answer
    E. All of the following are true
    Explanation
    All of the statements were indeed true. Let's take a look at them one at a time.

    Beta-1 receptors are located on the heart. These receptors are blocked so norepinephrine can't act on these receptors as well. Heart rate will decrease and (A) is true.

    In the kidney, renin release is greatly influenced by sympathetic beta-1 stimulation. (By the way, the other influence is the amount of sodium reaching the kidney and renal perfusion pressure). So blocking these receptors means less renin can be released. (B) is a correct statement. You can imagine that individuals with higher levels of renin should have a greater response to beta-blockers than those with lower levels of renin. Indeed, that is the case.

    Any receptor blocker of any kind will be more non-selective as you increase the dose. That is the problem here, too. The goal is to target beta-1 receptors on the heart, but too much blockade could mean that the agent can start blocking beta-2 and even beta-3 receptors, which can lead to some serious side effects. (C) is true.

    Exercise performance evaluation is tricky, but it can be understood. As contractility is decreased from beta-blockers, stroke volume is also decreased. The combined decreases in stroke volume and heart rate will decrease cardiac output. When exercising, muscles need blood flow and will therefore increase cardiac output to match this need. However, beta blockers will limit the extent of increasing the cardiac output, so exercise performance will never be as high as before the initiation of a beta-blocker. (D) is a correct statement.

    Rate this question:

  • 5. 

    Which of the following is true regarding calcium channel blockers?

    • A.

      Dihydropyridines are useful for the treatment of arrhythmias

    • B.

      Smooth muscle of veins are not significantly affected

    • C.

      They are useful in the treatment of coronary artery disease

    • D.

      B and C

    • E.

      All of the above

    Correct Answer
    D. B and C
    Explanation
    (D) is the most correct answer here.

    Calcium channel blockers (CCBs) receive their claim to fame for their ability to relax arteries and arterioles. They have no significant effect on the smooth muscle of veins. By binding to those calcium channels, they reduce intracellular calcium (since calcium won't be able to come into the cell from the extracellular space) and therefore reduces contraction. This relaxation will reduce the TPR and thereby reduce blood pressure. (B) is correct.

    Calcium channel blockers are pretty good with coronary artery disease. As mentioned before, CCBs will relax arteries, even the coronary arteries to increase blood flow to the heart. This will be discussed further on. Actually, CCBs are good in those with low renin levels to lower blood pressure because drugs that inhibit some part of the RAS will not be as effective in these individuals since there is already low renin levels. (C) is a correct statement.

    (A) is a false statement. This is because NON-dihydropyridines are useful for the treatment of arrythmias. Examples of these drugs are verapamil and diltiazem. They ultimately affect AV node conduction, a discussion for later.

    There is one other point to make about CCBs. They relax arterioles but not the veins. In the capillaries, where the two vessels meet, the arterioles will relax but not the veins, which will lead to an increase hydrostatic pressure in the capillary bed that could lead to peripheral edema.

    Rate this question:

  • 6. 

    Angiotensin converting enzyme (ACE) inhibitors are effective in treating hypertension because

    • A.

      They prevent vascular smooth muscle contraction

    • B.

      They prevent the secretion of aldosterone

    • C.

      They prevent the reabsorption of sodium in the nephron

    • D.

      A and B

    • E.

      All of the above

    Correct Answer
    E. All of the above
    Explanation
    (E) is the most correct answer here.

    (A) is true. ACE inhibitors will prevent the formation of angiotensin II, which would normally act on vascular smooth muscles to initiate a contraction. This formation is prevented, resulting in less contraction.

    (B) is true. Angiotensin II will lead to the secretion of aldosterone. Less angiotensin II, which is the case with an ACE inhibitor, will mean less aldosterone secreted by the kidney.

    (C) is true. Less aldosterone is secreted. Aldosterone would normally increase reabosprtion of sodium in the nephron, but with less aldosterone, there is less reabsorption.

    It must be noted that the fall in blood pressure due to less secretion of aldosterone takes time. So in the short-term, there may be a reduction in cardiac output due to increased sodium and water excretion. In the long-run, there is a reduced secretion of aldosterone that will then decrease TPR to lower blood pressure.

    When an ACE inhibitor is administered, TPR decreases quite early in therapy. This is from the prevention of angiotensin II and the subsequent relaxation of vascular smooth muscle. It needs to be clear that this is separate from aldosterone.

    Rate this question:

  • 7. 

    According to JNC 8 guidelines, which of the following is the blood pressure goal for a 70 year old male with diabetes?

    • A.

      < 140/90 mm Hg

    • B.

      < 130/90 mm Hg

    • C.

      < 140/80 mm Hg

    • D.

      < 130/80 mm Hg

    • E.

      < 150/90 mm Hg

    Correct Answer
    A. < 140/90 mm Hg
    Explanation
    According to the recently published JNC 8 report, new guidelines had stated that the more reasonable goal for individuals who are at least 65 years old AND have diabetes is < 140/90 mm Hg. For your information:

    (C) refers to the blood pressure goal by the American Diabetes Association (ADA).

    (D) refers to the blood pressure goal for individuals over the age of 65 and have diabetes, according to JNC 7.

    (E) actually refers to the blood pressure goal for those over the age of 60, according to JNC 8!

    Rate this question:

  • 8. 

    In which part of the renal tubule is sodium NOT reabsorbed?

    • A.

      Ascending Loop of Henle

    • B.

      Proximal tubule

    • C.

      Collecting Duct

    • D.

      Descending Loop of Henle

    • E.

      Distal tubule

    Correct Answer
    D. Descending Loop of Henle
    Explanation
    Answer (D) is correct. Remember, in the descending Loop of Henle, water will be the one that is absorbed. This is crucial because then that allows the luminal fluid ("urine") to be more concentrated as it goes down into the renal medulla. In all other places, sodium will be reabsorbed, especially in the proximal tubule. This is the site of major sodium reabsorption. The ascending Loop of Henle has some significant sodium reabsoprtion, too. The distal tubule is the "last" chance for sodium reabsorption and control at this portion is crucial for blood pressure control. Quotes are used because sodium is indeed reabsorbed in the collecting duct. That is truly the last resort for both water and sodium and has been targeted by drugs.

    Rate this question:

  • 9. 

    When a carbonic anhydrase inhibitor is administered, what is the effect on the pH of the urine?

    • A.

      Increase

    • B.

      Decrease

    • C.

      No effect

    Correct Answer
    A. Increase
    Explanation
    Carbonic anhydrase inhibitors, as the name suggests, will inhibit the enzyme carbonic anhydrase. The best way to think of this problem is to think through the mechanism.

    There is a Na-H antiporter in the renal tubule. As sodium comes from the urine into the cell, hydrogen will be pumped out. Within the cell, however, carbon dioxide and water combine to form carbonic acid (H2CO3) via carbonic anhydrase. Carbonic acid will split into hydrogen and bicarbonate. With an inhibitor of carbonic anhydrase, carbonic acid won't be formed and hydrogen will not be as plentiful in the cell. This means that the antiporter won't function as well because the gradient from the inside of the cell to the outside of the cell (the direction for the hydrogen ion) won't be as steep (less hydrogen ion in the cell as stated before). Ultimately, sodium stays in the urine, as does water, hence the diuretic. Now, on the luminal side, carbonic anhydrase will allow carbonic acid to go back to carbon dioxide and water. Yet that is also inhibited and so it will stay as bicarbonate and hydrogen ion. Bicarbonate appears more in the urine (there won't be an increase in hydrogen ion since it wasn't pumped out from the cell) and therefore the pH of the urine goes up.

    By the way, the pH of the blood will decrease and may result in metabolic acidosis.

    At the end of the day, sodium is lost, which is what you want. But why would you use this? You could probably use it to reduce intraocular pressure and for high altitude sickness (the metabolic acidosis stimulates respiration). You could even use it for metabolic alkalosis!

    Rate this question:

  • 10. 

    Which of the following is the correct mechanism of action for osmotic diuretics?

    • A.

      Blocks the Na-K-2Cl symporter

    • B.

      Blocks endogenous vasopressin receptors

    • C.

      Increases osmolarity of tubular fluid

    • D.

      Blocks Na-Cl symporter

    • E.

      Blocks aldosterone receptors

    Correct Answer
    C. Increases osmolarity of tubular fluid
    Explanation
    (C) explains the mechanism of action for an osmotic diuretic, as the name would suggest. Osmostic diuretics like mannitol will increase the osmolarity of tubular fluid and suck water into the tubular fluid for excretion. This is usually given IV because oral administration would lead to diarrhea. These diuretics are used to decrease intracranial pressure (suck water out of the brain) and to decrease intraocular pressure (suck water out of the eye).

    (A) is the mechanism for loop diuretics
    (B) is the mechanism for ADH antagonists
    (D) is the mechanism for thiazide diuretics
    (E) is the mechanism for aldosterone antagonists (potassium-sparing diuretics)

    Rate this question:

  • 11. 

    Which of the following is NOT a side effect of loop diuretics?

    • A.

      Hyperkalemia

    • B.

      Hyperuricemia

    • C.

      Hyponatremia

    • D.

      Hypotension

    • E.

      Volume depletion

    Correct Answer
    A. Hyperkalemia
    Explanation
    Loop diuretics are potent, so when they block reabosrption in the ascending Loop of Henle, they really block a lot of sodium reabsorption. So the sodium stays in the urine. However, the kidney will sense this and activate the RAS to secrete aldosterone. As noted many times, aldosterone will reabsorb sodium and push potassium out in exchange, leading to HYPOkalemia, which is why (A) is the correct answer.

    (B) is a little more subtle, but loop diuretics and uric acid are pumped into the tubule at the proximal tubule; they compete for the same transporter. This means that more uric acid will be left in the blood, which can lead to crystal formation and gout. Therefore, loop diuretics should be used in caution in those with gout.

    (C) is intuitive, because the goal of loop diuretics is to excrete sodium and water, so you can overshoot and lead to HYPOnatremia (too little sodium in blood).

    (D) is also intuitive. It is possible to overshoot and then reduce blood pressure too much, leading to hypotension.

    (E) makes sense because the point of diuretics is to lose sodium (and thus water). Losing water decreases volume and thus leads to volume depletion in extreme cases.

    Rate this question:

  • 12. 

    Which of the following statements is false?

    • A.

      Hydrochlorothiazide works in the distal tubule of the nephron

    • B.

      Loop diuretics and thiazide diuretics are each contraindicated with potassium-sparing diuretics due to the risk of severe hyponatremia

    • C.

      Vasopressin receptor antagonists cause sodium concentration to increase in the serum

    • D.

      Spironolactone may block other steroid receptors

    • E.

      Hydrochlorothiazide may be used in combination with angiotensin converting enzyme (ACE) inhibitors

    Correct Answer
    B. Loop diuretics and thiazide diuretics are each contraindicated with potassium-sparing diuretics due to the risk of severe hyponatremia
    Explanation
    (B) is the false statement because both of those types of diuretics CAN be used with potassium-sparing diuretics. As stated before, loop diuretics and thiazide diuretics can lead to hypokalemia from subsequent activation of the RAS. But a potassium-sparing diuretic will block the receptors for aldosterone to minimize that concern. Hyponatremia is not a concern either.

    (E) is especially interesting because it works similarly. Using an ACE inhibitor will allow the RAS effect to be diminished, so that is why you may see combination lisinopril/hydrochlorothiazide products, for instance.

    Rate this question:

  • 13. 

    Which of the following is a negative feedback relationship?

    • A.

      Urine volume and extracellular fluid volume

    • B.

      Cardiac output and blood pressure

    • C.

      Blood pressure and urine volume

    • D.

      Venous return and cardiac output

    Correct Answer
    A. Urine volume and extracellular fluid volume
    Explanation
    As urine output increases, the extracellular fluid volume has to decrease because all the urine output is said to be from the exctracellular volume.

    All the others are directly related. (B) is interesting because most individuals with hypertension actually have an increase in TPR rather than cardiac output, but the relationship still exists. (D) is noted because all of the blood that comes back to the heart (venous return) will be pumped out again (cardiac output).

    Rate this question:

  • 14. 

    In someone undergoing treatment for hypertension, why does blood pressure ultimately decrease to normal?

    • A.

      Metabolic autoregulation

    • B.

      Diuretic action on blood vessels

    • C.

      Cardiac output increases

    • D.

      A and C

    • E.

      All of the above

    Correct Answer
    A. Metabolic autoregulation
    Explanation
    (A) is correct here. Metabolic autoregulation is what allows sustained reponse due to chronic diuretic therapy. Arterioles will begin to dilate, which help lower TPR. This is important to counter the effects of the underlying reasons for why hypertension came about in the first place. It must be noted that the profile will look very similar to a normotensive individual.

    (B) is not true because diuretics do not act on blood vessels.

    (C) is not from the long-term perspective. In the short-term, cardiac output decreases shortly after beginning the diuretic because of the loss of extracellular fluid. In the long-term, metabolic autoregulation is king.

    Rate this question:

  • 15. 

    If a 65 year old male has a blood pressure of 140/86 mm Hg, what is his mean blood pressure?

    • A.

      158 mm Hg

    • B.

      104 mm Hg

    • C.

      110 mm Hg

    • D.

      95 mm Hg

    • E.

      113 mm Hg

    Correct Answer
    B. 104 mm Hg
    Explanation
    There are two ways to calculate this and they will both lead you to the right answer.

    Mean BP = (Systolic - Diastolic)/3 + Diastolic = (140-86)/3 + 86 = 104

    Mean BP = (2/3)Systolic + (1/3)Diastolic = (2/3)(140) + (1/3)(86) = 104

    Rate this question:

  • 16. 

    What effect does the rostral ventrolateral medulla have on blood pressure?

    • A.

      Increase

    • B.

      Decrease

    • C.

      No effect

    Correct Answer
    A. Increase
    Explanation
    Baroreceptors in the carotid sinus will detect blood pressure abnormalities and will then make changes. Activation of the rostral ventrolateral medulla (RVLM) requires lessening the inhibition. That means that it is normally inhibited. Indeed, the caudal ventrolateral medulla (CVLM) will send inhibitory transmitters to put a brake on the RVLM. This occurs when the baroreceptors detect a rise in blood pressure. This is the negative feedback loop. When the baroreceptors detect a rise in blood pressure, there will be less impulses coming from the RVLM, leading to a compensatory decrease in blood pressure, such as when going from standing to sitting. When there is a decrease in blood pressure, there is a smaller number of impulses to the baroreceptors and therefore less of an inhibition on the RVLM to ultimately increase the blood pressure. This is what happens when going from sitting to standing. Therefore, the RVLM will increase blood pressure.

    Rate this question:

  • 17. 

    What is the mechanism of action of alpha-methyldopa and clonidine?

    • A.

      Beta-1 agonist

    • B.

      Beta-1 antagonist

    • C.

      Alpha-1 agonist

    • D.

      Alpha-2 agonist

    • E.

      Alpha-2 antagonist

    Correct Answer
    D. Alpha-2 agonist
    Explanation
    (D) is the correct answer here. To figure out why, it is important to understand that on the RVLM, there are alpha-2 receptors. These receptors, when activated, will result in a decrease in the rate of depolarization of the RVLM neurons. This ultimately inhibits the RVLM from raising the blood pressure, which explains the antihypertensive effects.

    Alpha-2 receptors are actually one postganglionic sympathetic neurons that innervate the vascular smooth muscle. When activated, the receptors will lead to inhibition of norepinephrine release, and that leads to decreased effects of norepinephrine on the vascular smooth muscle (since norepinephrine would normally act on alpha-1 receptors on the muscle and therefore less contraction) and on the heart (since norepinephrine would normally act on beta-2 receptors on the heart and therefore less sympathetic tone).

    These drugs aren't really used too much anymore, but clonidine can be introduced in individuals who have uncontrolled blood pressure. Regarding clonidine in particular, there is a rebound phenomenon with sudden withdrawal that can result in a hypertensive crisis. This is somewhat expected because if you keep blocking norepinephrine release, the body will try to pump out more norepinephrine in vain. But if you suddenly remove the inhibition from clonidine, all that stored norepinephrine will be pumped out and "over"innervate.

    Rate this question:

  • 18. 

    If a ganglionic blocking agent were administered, heart rate would _______ and cardiac output will _______

    • A.

      Increase; increase

    • B.

      Increase; decrease

    • C.

      Decrease; decrease

    • D.

      Decrease; increase

    • E.

      There will be no effect on EITHER heart rate OR cardiac output

    Correct Answer
    B. Increase; decrease
    Explanation
    (B) is correct here. It is important to understand what ganglionic blocking agents do. These agents block nicotinic receptors that would ultimately innervate the sympathetic nervous system. Therefore, both the sympathetic AND the parasympathetic ganglia are blocked. You would then expect less sympathetic tone to the arteries (TPR decreases) and less sympathetic tone to veins (cardiac output decreases).

    What about heart rate? Quickening the heart rate is the hallmark of the sympathetic nervous system. Slowing down the heart rate is the hallmark of the parasympathetic nervous system. So which one wins? The answer is neither. In this case, the heart will beat at its intrinsic rate of 110 beats/min, which is much faster than normally at rest (60-80 beats/min). Therefore, the heart rate would actually increase when both systems are blocked.

    Rate this question:

  • 19. 

    Which of the following is true regarding alpha-blockers?

    • A.

      Orthostatic hypotension may be a concern

    • B.

      They are widely used to alleviate the symptoms of benign prostatic hyperplasia (BPH)

    • C.

      They decrease cardiac output and total peripheral resistance

    • D.

      A and B

    • E.

      All of the above

    Correct Answer
    E. All of the above
    Explanation
    (E) is the correct answer here. Let's go through each answer choice one by one, beginning with an overview of alpha-blockers.

    Alpha-blockers are agents that will block alpha-one receptors, which are on vascular smooth muscle cells. Normally, these receptors will constrict the arteries and constrict the veins. Blocking them, like these agents do, will relax both arteries and veins, thereby decreasing venous return and decreasing TPR.

    You can imagine that if you relax the veins too much, the blood will be pooled towards the feet and lead to orthostatic hypotension. Indeed, this is a concern, especially with the "first dose phenomenon", where the first dose was too large, leading to this condition. Remember, these are competitive antagonists, so start low to get the desired effect. Too much could lead to too much blocking and side effects. (A) is true.

    Prostate smooth muscle in men have alpha-one receptors and these agents will relax the smooth muscle in the enlarged prostate. Therefore, alleviation of symptoms (hopefully) will occur with these drugs. (B) is true.

    Decreased venous return indicates a decreased cardiac output, since they are directly related. Relaxation of veins decreases TPR. (C) is also a correct statement.

    Rate this question:

  • 20. 

    Which of the following is true regarding administration of a non-selective alpha-2 antagonist?

    • A.

      Cardiac output increases

    • B.

      Heart rate increases

    • C.

      Heart rate decreases

    • D.

      Norepinephrine secretion in the synapse decreases

    • E.

      There is no effect on vascular smooth muscle

    Correct Answer
    B. Heart rate increases
    Explanation
    (B) is the correct statement here. Let's think about it for a second.

    There are alpha-two receptors on the postganglionic sympathetic neuron. These normally would block the release of norepinephrine. These would be blocked so that norepinephrine release is uninhibited and can do as it pleases. However, on vascular smooth muscle, there are alpha-one receptors on it that norepinephrine will act on to cause constriction. Non-selective alpha-2 antagonists will block these, too. So despite the fact that there is more norepinephrine being released, there is no increased effect on the vascular smooth muscle, such as veins, because the receptors on which norepinephrine would act are blocked also!

    Yet there is still an effect on vascular smooth muscle. Veins will be relaxed anyway since there is less activation of those alpha-one receptors, so there is less venous return and less cardiac output. (A) is therefore false. (D) is false because secretion of norepinephrine will be increased due to less inhibition from the blocking of alpha-2. (E) is also false because there is indeed an effect on vascular smooth muscle.

    What about heart rate? Heart rate will increase and the explanation is as follows. There are still postganaglionic sympathetic neurons that have alpha-2 receptors. Classic. They are still blocked so there is more norepinephrine being released into the synapse. In the heart's sinoatrial node, which controls the heart rate, there are beta-1 receptors instead. Alpha-blockers won't touch beta receptors, so norepinephrine will greatly act on the beta-one. Remember, there is more norepinephrine out there to act on these beta-1 receptors, so the heart rate will increase and therefore (C) is false.

    Rate this question:

  • 21. 

    The following are possible side effects of beta-blockers EXCEPT

    • A.

      Fatigue

    • B.

      Hyperglycemia

    • C.

      Bronchospasm

    • D.

      Palpitations if withdrawn too quickly

    Correct Answer
    B. Hyperglycemia
    Explanation
    Beta-blockers are useful but can have some side effects.

    Fatigue can set in, especially with exercise. See the previous question's explanation as to why that is the case. (A) is true.

    Bronchospasm is a result of high doses of beta-blockers. Beta-2 receptors on bronchial smooth muscle that normally relax the smooth muscle. Beta-blockers can block these receptors and constrict the smooth muscle, leading to exacerbation of asthma or some pulmonary disease. Therefore, you might want to try some other class of antihypertensive drugs. (C) is true.

    Blocking beta-1 receptors on the heart would decrease heart rate. But the body senses this as not enough norepinephrine acting on receptors and will therefore generate more beta-1 receptors. This is fine if the beta-blocker is still present. Removing the beta-blocker suddenly means way too much norepinephrine will innervate the heart and dramatically increase the heart rate, which will lead to palpitations. It is advised that you should go off the beta-blocker slowly (taper off) rather than suddenly to prevent this. (D) is correct.

    Release of glucose requires stimulation of beta-2 receptors. Individuals taking insulin may lower blood sugar too much. That normally isn't a problem because the liver will pump out glucose to counteract this via stimulation of beta-2 receptors. Blocking these receptors may lead to less glucose being pumped out and therefore lead to hypoglycemia. As a result, (B) is the incorrect statement.

    Rate this question:

  • 22. 

    With regards to smooth muscle contraction, which of the following is true?

    • A.

      There is not sufficient calcium in the sarcoplasmic reticulum to initiate a contraction

    • B.

      Cacium is removed through a potassium-calcium exchange transporter

    • C.

      Norepinephrine and angiotensin II are agonists when opening the sarcoplasmic membrane calcium channels

    • D.

      A and C

    • E.

      All of the above

    Correct Answer
    D. A and C
    Explanation
    (D) is the most correct statement. Recall that for smooth muscle, there is not sufficient calcium in the sarcoplasmic reticulum for contraction so calcium from the extracellular space must be brought in to initiate a contraction. In the smooth muscle cell, the magnitude contraction is dependent on on the concentration of the calcium within the cell. (A) is a true statement.

    The sarcoplasmic reticulum will release calcium and is activated via two ways: electrical and agonist-initiated. An action potential can stimulate calcium channels on the sarcoplasmic reticulum and have it release calcium. Agonists like norepinephrine and angiotensin II will activate receptors on the smooth muscle cell to kick-start intracellular messenger reactions to ultimately release calcium from the sarcoplasmic reticulum. (C) is correct.

    (B) is an incorrect statement because calcium is removed from the cytoplasm via SODIUM-calcium exchange transporters. That also helps with the gradient for the Na-K-ATPase pumps.

    Rate this question:

  • 23. 

    Which of the following is true regarding hydralazine?

    • A.

      It interferes with the release of calcium from the sarcoplasmic reticulum

    • B.

      Reflex tachycardia is a concern

    • C.

      Can be combined with an inhibitor of the renin-angiotensin system

    • D.

      A and C

    • E.

      All of the above

    Correct Answer
    E. All of the above
    Explanation
    (E) is the correct statement here. Let's take a look why.

    Hydralazine's mechanism of action is indeed to interfere with the release of calcium from the sarcoplasmic reticulum. This will reduce the intracellular concentration of calcium and reduce contraction, reducing TPR and thus lowering blood pressure. This is actually a very potent vasodilator.

    The body will sense this blood pressure and will thus try to counter this. One of the ways it will do this is by releasing more norepinephrine into the heart to act on beta-1 receptors. This happens via the baroreceptor reflexes. Too much stimulation of the beta-1 receptors leads to an increase heart rate and palpitations. You would expect someone to be given a beta-blocker to stop this and that is indeed the case.

    Once again, the norepinephrine released will act on beta-1 receptors, even in the kidney to release renin. Therefore, the RAS is activated to compensate. All of the effects of the RAS apply here. So you may expect to combine hydralazine with an ACE inhibitor or even a diuretic for the sodium retention. Indeed, that is something that is done with hydralazine.

    Rate this question:

  • 24. 

    How does the mechanism of action of minoxidil lead to a decrease in blood pressure?

    • A.

      It opens the potassium channels to allow potassium to enter the smooth muscle cell, thereby depolarizing the cell

    • B.

      It opens the potassium channels to allow potassium to enter the smooth muscle cell, thereby hyperpolarizing the cell

    • C.

      It opens the potassium channels to allow potassium to exit the smooth muscle cell, thereby depolarizing the cell

    • D.

      It opens the potassium channels to allow potassium to exit the smooth muscle cell, thereby hyperpolarizing the cell

    Correct Answer
    D. It opens the potassium channels to allow potassium to exit the smooth muscle cell, thereby hyperpolarizing the cell
    Explanation
    Minoxidil will act on the potassium channels of smooth muscle cells. This will open the channel and allow potassium to EXIT the cell. This will then HYPERPOLARIZE the cell and the L-type calcium channels will in their closed resting state. Less calcium means less contraction and thereby lowering TPR.

    By the way, the baroreceptors will sense this and release norepinephrine. For compensation, the RAS will also be activated.

    Rate this question:

  • 25. 

    Which of the following is false regarding nitrates?

    • A.

      They relax smooth muscle

    • B.

      They decrease intracellular concentrations of nitric oxide

    • C.

      They are contraindicated with concurrent use of phosphodiesterase type 5 inhibitors

    • D.

      Long-term adminsitration of sodium nitroprusside may lead to cyanide poisoning

    • E.

      Nitrates have a short duration of action due to rapid metabolism

    Correct Answer
    B. They decrease intracellular concentrations of nitric oxide
    Explanation
    (B) is the false statement here. Nitrates actually increase intracellular concentrations of nitric oxide to provide vasodilation. Vasodilation will mean that the arteries are relaxed and the TPR will be reduced, thereby lowering blood pressure.

    (A) is true. Nitrates do indeed relax smooth muscle and help with vasodilation by metabolism into nitric oxide.

    (C) is a true statement. Phosphodiesterase type 5 inhibitors (sildenafil, tadalafil, etc.) will increase cyclic GMP levels for vasodilation. The problem is that there would be too much vasodilation with nitrates and that could lead to an unsafe drop in blood pressure. If you ever see those commercials for Viagra or Cialis, then you are familiar with the statement: "Do not take Viagra [or Cialis] if you take nitrates for chest pain as this could lead to an unsafe drop in blood pressure." This is because of the excessive vasodilation.

    Sodium nitroprusside is actually metabolized and lead to cyanide. In the short-term, it's no problem. However, long-term administration may lead to cyanide poisoning, which is extremely fatal so (D) is true.

    (E) is also true. A short duration of action is crucial, especially is someone takes a nitrate for chest pain (angina). Those individuals want instant relief and if you know someone who does take nitrates, then maybe they put it under their tongue before walking up a flight of stairs, for instance. Either way, a short duration of action is true.

    Rate this question:

  • 26. 

    Which of the following is false regarding the renin-angiotensin system (RAS)?

    • A.

      RAS will be activated when dietary sodium is low

    • B.

      The macula densa region of the nephron detects low plasma sodium concentration and signals the juxtaglomerular cells to release renin

    • C.

      Angiotensin II is less potent than angiotensin I

    • D.

      The release of renin is inversely proportional to renal perfusion pressure

    • E.

      Renin release is directly proportional to activity of the sympathetic nervous system

    Correct Answer
    C. Angiotensin II is less potent than angiotensin I
    Explanation
    (C) is the false statement here. Angiotensin II is actually MORE potent than angiotensin I and that is why many drugs have been developed to block this conversion via angiotensin converting enzyme (ACE).

    (A) is true because when sodium intake is low, there is less sodium in the tubular fluid and thus less will be sensed by the kidney. So the kidney will activate thee RAS to reabsorb more sodium.

    (B) is true. This is just a fact, but the idea is that this region (which is situated right before the distal tubule) senses the low sodium and tells the juxtaglomerular cells to release renin.

    (D) is true. Blood pressure in the kidney is sensed by renal baroreceptors. If it is low, it is sensed as low volume (and thus low sodium) so the kidney will release renin in response. Higher pressure will mean lower amounts of renin. In individuals with atherosclerosis, especially in the afferent arteriole, there is less pressure after the obstruction. This will release renin anyway and increase the pressure coming in. This is already a problem before the obstruction and will significantly increase the pressure before the obstruction. This is termed "renovascular hypertension".

    (E) is also true. Sympathetic neurons will activate the juxtaglomerular cells, since they have beta-1 receptors. The RVLM will fire in response to a low blood pressure, thereby increasing neural activity to the kidney. Renin is released and angiotensin II is formed to therefore increase the blood pressure back to the set point.

    Rate this question:

  • 27. 

    Which of the following is/are side effects of ACE inhibitors?

    • A.

      Dry cough

    • B.

      Hyperaldosteronism

    • C.

      Hypokalemia

    • D.

      A and C

    • E.

      All of the above

    Correct Answer
    A. Dry cough
    Explanation
    (A) is the only correct answer here. Dry cough is reported by 5-20% of patients that may result in discontinuation of therapy.

    (B) is false. Hyperaldosteronism means that there is too much aldosterone in the bloodstream. With an ACE inhibitor, the secretion of aldosterone is blocked so there would actually be LESS aldosterone in the blood.

    (C) is also false. Aldosterone normally would activate sodium reabsorption in the kidney by exchanging with potassium; potassium is lost. However, less aldosterone is secreted so less sodium is reabsorbed, which means less potassium is excreted. Therefore, more potassium stays in the blood and may lead to HYPERkalemia.

    Something to note is that ACE inhibitors have a rare but serious side effect of angioedema. This is swelling of the tongue and larynx that may be life-threatening. ACE inhibitors are actually pretty useful in the treatment of heart failure, which will be discussed later.

    Rate this question:

  • 28. 

    Which of the following is false when comparing ACE inhibitors and angiotensin receptor blockers (ARBs)?

    • A.

      ACE inhibitors may have a dry cough as a side effect that is not as prevalent with ARBs

    • B.

      ACE inhibitors block an enzyme whereas ARBs block a receptor site.

    • C.

      Both classes of drugs can be combined with an antihypertensive of another class

    • D.

      Only ARBs will help to lower total peripheral resistance

    • E.

      In the long-run, both classes of drugs will reduce the secretion of aldosterone.

    Correct Answer
    D. Only ARBs will help to lower total peripheral resistance
    Explanation
    (D) is the false statement. Both classes of drugs will help to lower TPR, but they do so differently.

    (A) is true. The side effect of dry cough can be too bothersome for some patients so a great alternative is an ARB, which does NOT have this side effect as much as ACE inhibitors.

    (B) is true. ACE inhibitors inhibit the angiotensin converting enzyme to prevent the formation of angiotensin II, and ARBs block the angiotensin II receptors on vascular smooth muscle.

    (C) is also true. Diuretics are notorious for compensatory sodium reabsorption. In those cases, it is efficacious to block the effects of aldosterone by inhibiting the RAS. Indeed, the diuretic hydrochlorothiazide is often combined with lisinopril, an ACE inhibitor, for this purpose. Hydrochlorothiazide is also combined with losartan for this same reason. It must be noted that these drugs, when combined with potassium-sparing diuretics (spironolactone, eplerenone, etc.) could lead to significant hyperkalemia.

    (E) is true. As discussed for ACE inhibitors, ARBs will eventually decrease the secretion of aldosterone, but that takes time to do.

    For the most part, ACE inhibitors and ARBs are very similar in action and efficacy.

    Rate this question:

  • 29. 

    In the renal tubule where sodium is reabsorbed, which of the following is common throughout the nephron?

    • A.

      Na-K-ATPase transporter

    • B.

      Na-Cl symporter

    • C.

      Na-K-2Cl symporter

    • D.

      Na-H antiporter

    • E.

      A and D

    Correct Answer
    A. Na-K-ATPase transporter
    Explanation
    (A) is the correct answer. This is such an important concept. In the renal tubule, the kidney tries very hard to maintain volume by reabsorbing sodium (and thus water). It will do this even at the expense of potassium. It is so vital to maintain volume that it needs the capacity to do so at every step of the process. This transporter is all over the body and the renal tubule is no exception. It is this transporter than can lead to various electrolyte abnormalities.

    (B) is located primarily in the distal tubule.

    (C) is located primarily in the ascending loop of Henle.

    (D) is located primarily in the proximal tubule.

    Rate this question:

Quiz Review Timeline +

Our quizzes are rigorously reviewed, monitored and continuously updated by our expert board to maintain accuracy, relevance, and timeliness.

  • Current Version
  • Mar 22, 2023
    Quiz Edited by
    ProProfs Editorial Team
  • Dec 20, 2013
    Quiz Created by
    Cardtrick
Back to Top Back to top
Advertisement
×

Wait!
Here's an interesting quiz for you.

We have other quizzes matching your interest.